Cho tam giác abc có k là tâm đường tròn bàng tiếp góc a. biết 0 bkc bac 60 . hỏi bac bằng bao nhiêu?


Bắt đầu bởi baopbc, 23-05-2016 - 18:53

hình học


Chủ đề này có 434 trả lời

Đã gửi 09-06-2016 - 13:09

$\boxed{\text{Bài toán 29.}}$ Cho tam giác $ABC$ nội tiếp $[O]$ với đường đối trung $AD \;[D\in BC]$. Các điểm $E, F$ lần lược nằm trên $AC, AB$ sao cho $DE || AB$ và $DF || AC$. Gọi $A'$ là tâm đường tròn ngoại tiếp tứ giác $[BCEF]$. Xác định tương tự các điểm $B', C'$. Chứng minh rằng $AA', BB', CC'$ đồng quy.

Nguồn: Nhớ là trong một cuốn sách nào đó.


Bài viết đã được chỉnh sửa nội dung bởi baopbc: 09-06-2016 - 13:09

  • quanghung86, halloffame, Element hero Neos và 4 người khác yêu thích

Quyết tâm off dài dài cày hình, số, tổ, rời rạc.

Đã gửi 09-06-2016 - 14:09

$\boxed{\text{Lời giải bài 29}}$ Gọi $T$ là giao điểm tiếp tuyến tại $B$, $C$ của $[O]$ và $L$ là điểm Lemoine của $\triangle ABC$. Ta có $[ADLT]=-1$ và $O, A', T$ thẳng hàng. Ta có $\widehat{BA'T}=\widehat{BEC}$ và $\widehat{BOA'}=\widehat{BAE}$ nên $\triangle BAE \sim \triangle BOA'$. Do đó $\frac{OA'}{OT}=\frac{OA'}{OB}.\frac{OB}{OT}=\frac{AE}{AB}.\cos{BAC}$. Đặt $BC=a, CA=b, AB=c$. Ta tính được $\frac{DB}{DC}=\frac{c^2}{b^2}$, suy ra $\frac{AE}{AC}=\frac{BD}{BC}=\frac{c^2}{c^2+b^2}$. Do đó $\frac{OA'}{OT}=\frac{bc}{b^2+c^2}.\frac{b^2+c^2-a^2}{2bc}=\frac{b^2+c^2-a^2}{2[b^2+c^2]}$. Mặt khác $[ADLT]=-1$, dùng đại số ta suy ra $[DTLA]=0,5$ do đó $\frac{LD}{LT}=\frac{AD}{2AT}$. Không khó để tính được tỉ số $\frac{AD}{AT}=\frac{b^2+c^2-a^2}{b^2+c^2}$. Do đó $\frac{LD}{LT}=\frac{OA'}{OT}$ hay $OL \parallel A'D$. Do $[ADLT]=-1$ nên $AA'$ chia đôi $OL$. Tương tự ta có đpcm.

P/s: bạn nào đề xuất bài mới giùm mình.

$$\begin{array}{| l | l |} \hline Ngockhanh99k48 & 5\\ \hline IHateMath & 1\\ \hline fatcat12345 & 4\\ \hline dogsteven & 5\\ \hline baopbc & 7\\ \hline QuangDuong12011998 & 2\\ \hline xuantrandong & 2\\ \hline mrjackass & 1\\ \hline vietnaminmyheart & 2\\ \hline BuiBaAnh & 1\\ \hline halloffame & 2\\ \hline\end{array}$$


Bài viết đã được chỉnh sửa nội dung bởi baopbc: 09-06-2016 - 14:40

Đã gửi 09-06-2016 - 14:20

$\boxed{\text{Bài toán 29.}}$ Cho tam giác $ABC$ nội tiếp $[O]$ với đường đối trung $AD \;[D\in BC]$. Các điểm $E, F$ lần lược nằm trên $AC, AB$ sao cho $DE || AB$ và $DF || AC$. Gọi $A'$ là tâm đường tròn ngoại tiếp tứ giác $[BCEF]$. Xác định tương tự các điểm $B', C'$. Chứng minh rằng $AA', BB', CC'$ đồng quy.

Nguồn: Nhớ là trong một cuốn sách nào đó.

 Mọi người chém nhiệt tình cho  topic nó sôi nổi đi nào 

Cách khác không cần tính toán:

Trước hết việc chứng minh $[BCEF]$ là tứ giác nội tiếp khá đơn giản nên em không nêu lại đây nữa

Gọi $L$ là điểm $Lemoine$ của tam giác $ABC$. $O$ là tâm đường tròn ngoại tiếp tam giác $ABC$

Qua $L$ kẻ đường thẳng song song $BC$ cắt $AB,AC$ lần lượt tại $B_1,C_1$

        $L$ kẻ đường thẳng song song với $AC$ và $AB$ cắt $AB$ và $AC$ lần lượt tại $B_2$ và $C_2$

Khi đó thì $B_1,C_1,B_2,C_2$ cùng nằm trên đường tròn $Lemoine$ thứ nhất của tam giác. Gọi $X$ là tâm đường tròn này

Khi đó $X$ là trung điểm $OL$

Xét phép vị tự tâm $A$ tỉ số $\dfrac{AL}{AD}$ biến

$B \rightarrow B_1$

$C \rightarrow C_1$

$F \rightarrow B_2$

$E \rightarrow C_2$

Suy ra $A' \rightarrow X$. Khi đó thì $AA'$ đi qua $X$ là trung điểm $OL$

Chứng minh tương tự thì suy ra $AA',BB',CC'$ đồng quy tại trung điểm $X$ của $OL$ $\blacksquare$

$$\begin{array}{| l | l |} \hline Ngockhanh99k48 & 5\\ \hline IHateMath & 1\\ \hline fatcat12345 & 4\\ \hline dogsteven & 5\\ \hline baopbc & 7\\ \hline QuangDuong12011998 & 2\\ \hline xuantrandong & 2\\ \hline mrjackass & 1\\ \hline vietnaminmyheart & 3\\ \hline BuiBaAnh & 1\\ \hline halloffame & 2\\ \hline\end{array}$$


Bài viết đã được chỉnh sửa nội dung bởi baopbc: 09-06-2016 - 14:40

  • canhhoang30011999, quanghung86, ineX và 3 người khác yêu thích

Đã gửi 09-06-2016 - 14:38

Một cách khác cho bài 29, dài hơn một chút!

$\boxed{\text{Lời giải bài toán 29}}$ [Cách này có lẽ phù hợp với mấy đứa chưa học biến hình như em]

$\boxed{\text{Bổ đề 1}}$ Cho tam giác $ABC,L$ là điểm Miquel. $M$ là trung điểm $BC.$ Đường cao $AH$. Khi đó $ML$ chia đôi $AH$.

Bổ đề trên là bổ đề quen thuộc về điểm Lemoine, mình học từ anh viet nam in my heart, về chứng minh mọi người có thể nhắn tin qua anh viet nam in my heart hoặc đọc trong các tài liệu về điểm Lemoine.

Giải bài toán. [Có lẽ vẫn cần tính toán nhưng không dài như anh Ngockhanh99k48]

Dễ thấy chỉ cần chứng minh $AA'$ đi qua trung điểm $OL$.

$T\equiv AD\cap OA'$. Theo tính chất quen thuộc, dễ thấy $[AD,LT]=-1.N,P$ theo thứ tự là trung điểm $AH,AD$. Do $AFDE$ là hình bình hành nên $P$ là trung điểm $EF\Longrightarrow A'P\perp EF\Longrightarrow AO\parallel A'P$. Áp dụng bổ đề 1 ta suy ra $M,L,N$ thẳng hàng. Mặt khác do $[AD,LT]=-1$ nên theo định lí Thales ta suy ra $A'D\parallel OL$. Mặt khác $A'[DA,LO]=-1\Longrightarrow AA'$ đi qua trung điểm $OL.\blacksquare$

Bài này không cần tính điểm đâu nhé!

Anh viet nam in my heart đề xuất bài mới đi!


Bài viết đã được chỉnh sửa nội dung bởi baopbc: 09-06-2016 - 14:39

Đã gửi 09-06-2016 - 14:45

Em xin đề xuất bài toán mới như sau:

$\boxed{\text{Bài toán 30}}$[Sưu tầm] Cho tam giác $ABC$ ngoại tiếp $[I]$, nội tiếp $[O]$. Tiếp điểm của $[I]$ trên các cạnh $BC,CA,AB$ lần lượt là $D,E,F$. $EF$ cắt đường tròn $[O]$ tại $D_1,D_2$. Tương tự ta xây dựng các điểm $E_1,E_2,F_1,F_2$. Gọi $X$ là tâm đẳng phương của $[DD_1D_2],[EE_1E_2],[FF_1F_2]$. Chứng minh rằng $X$ nằm trên $OI$


  • canhhoang30011999, quanghung86, ineX và 2 người khác yêu thích

Đã gửi 09-06-2016 - 16:42

$\boxed{\text{Lời giải bài toán 30}}$

Gọi $X'$ là trực tâm tam giác $DEF$.

Ta có $OI$ là đường thẳng Euler của tam giác $DEF$ nên $X' \in OI$.

Gọi $S=EF \cap BC$, $M_a, M_b, M_c$ là trung điểm $BC, CA, AB$.

Ta có $[BCA'S]=-1$ nên $SM_a.SD=SB.SC=SD_1.SD_2$ suy ra $M_a \in [DD_1D_2]$, tương tự $M_b \in [EE_1E_2]$,$M_c \in [FF_1F_2]$.

Gọi $O_D, O_E, O_F$ là tâm của $[DD_1D_2],[EE_1E_2],[FF_1F_2]$ thì $O_D$ là giao của trung trực $DM_a$ và trung trực $D_1D_2$.

Suy ra nếu gọi $D'$ là giao của $ID$ và trung trực $D_1D_2$ thì $O_D$ là trung điểm $OD'$,

tương tự $O_E$ là trung điểm $OE'$, $O_F$ là trung điểm $OF'$.

Ta lại có $DD', EE', OX'$ đồng quy tại $I$, $X'E \parallel OE'$, $X'D \parallel OD'$ nên $O_DO_E \parallel D'E' \parallel DE$.$[1]$

Lại có $FD_1.FD_2=FE_1.FE_2$ nên trục đẳng phương của $[DD_1D_2], [EE_1E_2]$ đi qua $F$.$[2]$

Từ $[1]$ và $[2]$ suy ra trục đẳng phương của $[DD_1D_2], [EE_1E_2]$ là đường cao qua $F$ của $\Delta DEF$.

Tương tự trục đẳng phương của $[EE_1E_2],[FF_1F_2]$ là đường cao qua $D$ của $\Delta DEF$,

                trục đẳng phương của $[FF_1F_2],[DD_1D_2]$ là đường cao qua $E$ của $\Delta DEF$,

Vậy $X \equiv X'$. [đpcm]

$\boxed{\text{Bài toán 31}}$[Thầy Trần Quang Hùng] Cho $\Delta ABC$ nội tiếp $[O]$, tâm nội tiếp $I$ và trực tâm $H.$ $A_1,A_2 \in BC$ sao cho $IA_1 \perp IB, IA_2 \perp IC.$ $AA_1,$ $AA_2$ cắt $[O]$ tại $A_3,$ $A_4.$ $A_3A_4$ cắt $BC$ tại $A_0.$ $B_0$ và $C_0$ được dựng tương tự. Chứng minh $A_0, B_0, C_0$ cùng thuộc đường thẳng vuông góc $IH.$

$$\begin{array}{| l | l |} \hline Ngockhanh99k48 & 5\\ \hline IHateMath & 1\\ \hline fatcat12345 & 5\\ \hline dogsteven & 5\\ \hline baopbc & 7\\ \hline QuangDuong12011998 & 2\\ \hline xuantrandong & 2\\ \hline mrjackass & 1\\ \hline vietnaminmyheart & 3\\ \hline BuiBaAnh & 1\\ \hline halloffame & 2\\ \hline\end{array}$$


Bài viết đã được chỉnh sửa nội dung bởi baopbc: 09-06-2016 - 22:07

Đã gửi 09-06-2016 - 21:52

Đành trâu bò vậy.

Kí hiệu $a$, $b$, $c$ là độ dài $BC$, $CA$, $AB$. $p,R,r$ là nửa chu vi, bán kính đường tròn ngoại tiếp, nội tiếp $\triangle ABC$.

$S_A=\dfrac{b^2+c^2-a^2}{2}$, $S_B=\dfrac{c^2+a^2-b^2}{2}$, $S_C=\dfrac{a^2+b^2-c^2}{2}$

các công thức sử dụng xem tại //blogcuaquang...oa-o-ti-cu.html

\[BA_1=\dfrac{IB}{\cos\dfrac{B}{2}}=\dfrac{2r}{2\sin\dfrac{B}{2}\cos\frac{B}{2}}=\dfrac{2r}{\sin B}=\dfrac{4Rr}{CA}=\dfrac{2ca}{a+b+c}=\dfrac{ca}{p}\]

\[\Rightarrow CA_1=a-BA_1=a-\dfrac{2ca}{a+b+c}=\dfrac{a[p-c]}{p}\]

\[\Rightarrow \dfrac{\overline{A_1B}}{\overline{A_1C}}=-\dfrac{c}{p-c}\]

Tương tự

\[\dfrac{\overline{A_2B}}{\overline{A_2C}}=-\dfrac{p-b}{p}\]

Tọa độ tỉ cự $A_1=[0:p-c:c]$, $A_2=[0:b:p-b]$. $AA_1$, $AA_2$ cắt $[ABC]$ tại $A_3$, $A_4$ thì

$A_3=\Big{[}-\dfrac{a^2[p-c]}{b^2-c^2+pc}: p-c:c\Big{]}$, $A_4=\Big{[}-\dfrac{a^2[p-b]}{c^2-b^2+pb}:b: p-b\Big{]}$. Từ đó lập phương trình đường thẳng $A_3A_4$

\[x\cdot p[p-a]+y\cdot a^2[p-b]\Big{[}\frac{c}{c^2-b^2+pb}-\frac{p-c}{b^2-c^2+pc}\Big{]}+z\cdot a^2[p-c]\Big{[}\dfrac{b}{b^2-c^2+pc}-\dfrac{p-b}{c^2-b^2+pb}\Big{]}=0\]

\[\Leftrightarrow x\cdot p[p-a]+y\cdot a^2[p-b]\dfrac{pb[b+c-p]}{[c^2-b^2+pb][b^2-c^2+pc]}+z\cdot a^2[p-c]\dfrac{pc[b+c-p]}{[c^2-b^2+pb][b^2-c^2+pc]}\]

$A_3A_4$ cắt $BC$ tại $A_0$ thì $A_0=[0:-c[p-c]:b[p-b]]$

Tương tự $B_0=[c[p-c]:0:-a[p-a]]$, $C_0=[-b[p-b]:a[p-a]:0]$

Do vậy $A_0$, $B_0$, $C_0$ thẳng hàng và đường thẳng chứa chúng có phương trình

\[a[p-a]x+b[p-b]y+c[p-c]z=0\]

$I=[a:b:c]$, $H=[S_BS_C:S_CS_A:S_AS_B]$. $IH$ có phương trình

\[x\cdot S_A[p-a][b-c]+y\cdot S_B[p-b][c-a]+z\cdot S_C[p-c][a-b]=0\]

Điểm vô cùng của $\overline{A_0.B_0,C_0}$ có tọa độ $[[b-c][p-a]:[c-a][p-b]:[a-b][p-c]]$ là điểm $X_{522}$ trong danh sách ETC //faculty.evans...opedia/ETC.html và điểm vô cùng của $IH$ chính là điểm $X_{515}$. Và cũng theo danh sách ETC[phần về $X_{515}$] thì có viết "$X_{515}$=orthopoint of $X_{522}$". Nghĩa của orthopoint xem tại //faculty.evans...a/glossary.html : khái niệm này chỉ dùng cho 2 điểm vô cùng, giả sử gọi $P$, $Q$, nếu $P$ nằm trên họ đường thẳng có phương $\ell_P$, $Q$ nằm trên họ đường thẳng có phương $\ell_Q$ thì $P$ gọi là orthopoint của $Q$ nếu $\ell_P\perp \ell_Q$.

Vậy kết luân $IH$ vuông góc $\overline{A_0,B_0,C_0}$.

P.S: Bài của thầy Hùng đã có tại //artofproblems...1230964p6219712 và Luis Gonzales, Telv Cohl có những lời giải khéo léo, đơn giản hơn.

Ngoài ra, nếu thay $I$ bởi điểm bất kì thì không phải lúc nào cũng đúng. Thay $I$ bởi $P$ thì $A_0$, $B_0$, $C_0$ chỉ thẳng hàng khi $P$, đẳng giác của $P$ và trọng tâm $\triangle ABC$ thẳng hàng.

Mình đề nghị bài tiếp theo

$\boxed{\text{Bài toán 32}}$[Quan sát được] Hai điểm $P$, $Q$ thỏa mãn $[PB,PC]+[QB,QC]=[PC,PA]+[QC,QA]=[PA,PB]+[QA,QB]=0$

$P_a$, $P_b$, $P_c$ đối xứng $P$ qua $BC$, $CA$, $AB$.

$Q_a$, $Q_b$, $Q_c$ đốii xứng $Q$ qua $BC$, $CA$, $AB$.

thì $[ABC]$, $[P_aP_bP_c]$, $[Q_aQ_bQ_c]$ có trục đẳng phương chung - nói cách khác là đồng trục.

$$\begin{array}{| l | l |} \hline Ngockhanh99k48 & 5\\ \hline IHateMath & 1\\ \hline fatcat12345 & 5\\ \hline dogsteven & 5\\ \hline baopbc & 7\\ \hline QuangDuong12011998 & 3\\ \hline xuantrandong & 2\\ \hline mrjackass & 1\\ \hline vietnaminmyheart & 3\\ \hline BuiBaAnh & 1\\ \hline halloffame & 2\\ \hline\end{array}$$


Bài viết đã được chỉnh sửa nội dung bởi baopbc: 09-06-2016 - 22:08

  • canhhoang30011999, quanghung86, ineX và 2 người khác yêu thích

Đã gửi 10-06-2016 - 10:47

$\boxed{\text{Lời giải bài 32}}$

Theo giả thiết, hai điểm $P, Q$ thỏa mãn tính chất trên được gọi là Antigonal conjugate. Ta có một tính chất rất đặc biệt của hai điểm liên hợp trên là liên hợp đẳng giác của $P, Q$ đối với $\triangle ABC$ là ảnh của nhau qua phép nghịch đảo đường tròn $[O]$.

Tính chất trên được đính kèm trong file dưới đây, mọi người có thể tham khảo

.

Quay lại bài toán trên, một điều hiển nhiên là tâm đường tròn $[P_aP_bP_c]$ là liên hợp đẳng giác của $P$ đối với $\triangle ABC$. Gọi $R, S$ thứ tự là liên hợp đẳng giác của $P, Q$ đối với $\triangle ABC$. Thế thì $R, S$ thứ tự là tâm $[P_aP_bP_c]$ và $[Q_aQ_bQ_c]$. Theo tính chất trên thì $R, S$ là ảnh của nhau qua phép nghịch đảo đường tròn $[O]$. Do đó $O, R, S$ thẳng hàng.

Gọi $\triangle R_aR_bR_c$ và $\triangle S_aS_bS_c$ là tam giác pedal ứng với $R, S$ đối với $\triangle ABC$.

Gọi $N_a, N_b, N_c$ là tâm Euler của $\triangle BPC, \triangle CPA, \triangle APB$.

Theo tinh chất 1 và tính chất 3 ở đường link: //artofproblems..._9point_centers, ta có $\triangle R_aR_bR_c \sim \triangle N_aN_bN_c \sim \triangle S_aS_bS_c$. Do đó nếu gọi $R_{[X]}$ là bán kính đường tròn $[X]$ thì :

$\frac{R_{[P_aP_bP_c]}}{R_{[Q_aQ_bQ_c]}} = \frac{2R_{[R_aR_bR_c]}}{2R_{[S_aS_bS_c]}} = \frac{R_bR_c}{S_bS_c} = \frac{AR}{AS}$. 

Do đó $\frac{OR}{OS} = \frac{AR^2}{AS^2} = \frac{AR^2 - R_{[P_aP_bP_c]}^2}{AS^2 - R_{[Q_aQ_bQ_c]}^2} = \frac{P_{A/[P_aP_bP_c]}}{P_{A/[Q_aQ_bQ_c]}}$. Theo định nghĩa về tỉ số phương tích ta có $[O, OA]$ đồng trục với $[P_aP_bP_c]$ và $[Q_aQ_bQ_c]$. Ta có đpcm $\blacksquare$. 

P/s: bài này nếu không biết nhiều cũng 'khó nhằn" chứ chẳng đùa. 

Mình xin đề nghị bài 33:

Giữ lại các điểm ở Bài đề nghị 1 của baopbc. Chứng minh $[F, FK]$ tiếp xúc $[O]$. 

Chú ý: Mong Bảo chỉ cộng điểm cho ai có lời giải thuần khiết thôi nhé.

$$\begin{array}{| l | l |} \hline Ngockhanh99k48 & 6\\ \hline IHateMath & 1\\ \hline fatcat12345 & 5\\ \hline dogsteven & 5\\ \hline baopbc & 7\\ \hline QuangDuong12011998 & 3\\ \hline xuantrandong & 2\\ \hline mrjackass & 1\\ \hline vietnaminmyheart & 3\\ \hline BuiBaAnh & 1\\ \hline halloffame & 2\\ \hline\end{array}$$

File gửi kèm

  •  Poncelet_point_and_its_applications_3.pdf   490.35K   1232 Số lần tải

Bài viết đã được chỉnh sửa nội dung bởi baopbc: 15-06-2016 - 10:39

  • quanghung86, halloffame, ineX và 2 người khác yêu thích

Đã gửi 11-06-2016 - 11:25

Cho những ai lười bật lại bài 1 của baopbc:

$\boxed{\text{Bài toán 33}}$ Cho tam giác $ABC$ có $\angle A=60^\circ,\angle B=80^\circ$ nội tiếp $\odot [O]$, ngoại tiếp $\odot [I]$.

$AI$ cắt $OB$ tại $X$, cắt $BC$ tại $M$. Đường thẳng qua $C$ vuông góc với $AB$ cắt $OM$ tại $K$ và trung trực $BC$ tại $E.ME$ cắt $AC$ tại $L$. Đường thẳng qua $K$ vuông góc với $BC$ cắt $AC$ tại $F$. Chứng minh $[F;FK]$ tiếp xúc $[O].$


Bài viết đã được chỉnh sửa nội dung bởi halloffame: 11-06-2016 - 11:32

  • quanghung86fatcat12345 thích

Sự học như con thuyền ngược dòng nước, không tiến ắt phải lùi.

Đã gửi 11-06-2016 - 11:54

$\boxed{\text{Lời giải bài toán 33}}$. Theo lời giải $\boxed{\text{Bài toán 1}}$ của anh Ngockhanhk9948 thì $\angle OMA=40^\circ$.

Mặt khác $\angle AMB=180^\circ-80^\circ-30^\circ=70^\circ\Longrightarrow \angle OMC=70^\circ\Longrightarrow \angle OKF=20^\circ$.

Lại có $\angle AOM=180^\circ-\angle MAO-\angle OMA=180^\circ-20^\circ-40^\circ=120^\circ$. Mà theo lời giải $\boxed{\text{Bài toán 1}}$ của anh dogsteven thì $\angle AOI=60^\circ\Longrightarrow \angle KOI=60^\circ$ hay $\triangle OIK$ đều.

Đường thẳng qua $K$ vuông góc với $OI$ cắt $OF$ tại $T$. Dễ thấy $\angle FKT=\angle 30^\circ-20^\circ=10^\circ\Longrightarrow \triangle TFK$ cân tại $F$ hay $T$ thuộc $[F,FK]$. Mặt khác theo lời giải $\boxed{\text{Bài toán 1}}$ của anh I Hate Math thì $\angle OSK=20^\circ$ với $S$ là điểm chính giữa cung $BC$ không chứa $A$ nên $T$ thuộc $[O]$. Mặt khác $T,F,O$ thẳng hàng nên $[F,FK]$ tiếp xúc $[O].\blacksquare$.

Bài toán đề nghị. $\boxed{\text{Bài toán 34}}$. [AopS]. Cho hình thang cân $ABCD$ nội tiếp $[O].DO$ cắt $BC$ tại $X$. Đường thẳng qua $X$ song song với $OA$ cắt $AB,AC$ theo thứ tự tại $E,F.[O']\equiv [DEF]$. Tiếp tuyến tại $A$ của $[O]$ cắt $[O']$ tại $P,Q$. Chứng minh rằng $[O'PQ]$ tiếp xúc $[O]$.

$\textbf{Hình vẽ bài toán}$

$$\begin{array}{| l | l |} \hline Ngockhanh99k48 & 6\\ \hline IHateMath & 1\\ \hline fatcat12345 & 5\\ \hline dogsteven & 5\\ \hline baopbc & 8\\ \hline QuangDuong12011998 & 3\\ \hline xuantrandong & 2\\ \hline mrjackass & 1\\ \hline vietnaminmyheart & 3\\ \hline BuiBaAnh & 1\\ \hline halloffame & 2\\ \hline\end{array}$$


Bài viết đã được chỉnh sửa nội dung bởi baopbc: 15-06-2016 - 10:40
Đề nghị bài toán mới

  • quanghung86fatcat12345 thích

Đã gửi 11-06-2016 - 13:41

Với bài này thì mình không dùng góc định hướng như mọi khi nữa, dựa theo hình vẽ trên của Bảo, khi mà đáy $AD$ bé hơn đáy $BC$.

Thực ra bài này chỉ cần chứng minh $[DEF]$ trực giao $[O]$.

$AB$ cắt $CD$ tại $T$. $DE$ cắt $OA$ tại $K$. $EF$ cắt $AD$ tại $J$.

Áp dụng định lý Thales

\[\dfrac{\overline{KA}}{\overline{KO}}=\dfrac{\overline{EJ}}{\overline{EX}}=\dfrac{\overline{EA}}{\overline{EB}}\]

theo định lý Thales đảo thì $DE$ song song $OB$.

Từ các đường thẳng song song này mà ta rút ra tính chất : $EA$ là phân giác $\angle DEF$.

Tiếp theo ta chỉ ra $A$ là tâm đường tròn nội tiếp $\triangle DEF$.  Chú ý là nếu $I$ là tâm nội tiếp $\triangle ABC$ thì $\angle BIC=90^o+\angle BAC$

\[\angle DEF=2\angle AEF=2\angle OAD\]

\[\angle DAF=180^o-\angle CAD=180^o-\angle ACB\Rightarrow 2\angle DAF=360^o-2\angle CAD=360^o-\angle AOD=180^o+2\angle OAD\]

Vậy $A$ là tâm nội tiếp $\triangle DEF$.

Mà $T$ thuộc phân giác $EA$, $TA=TD$ nên cũng có $TA=TD=TF$ và $T$ thuộc $[DEF]$.

\[\angle ODO'=\angle ODA+\angle O'DA=\angle ODA+\angle O'DE-\angle ADE=\angle ODA+90^o-\angle DTE-[\angle EAF-90^o]\]

\[=\angle ODA+90^o-\angle DTA+90^o-[180^o-\angle BAC]=\angle ODA+180^o-\angle DTA-180^o+\angle BAC\]

\[=\angle ODA+\angle BAC-\angle DTA=90^o-\dfrac{\angle AOD}{2}+\angle BAC-[\angle BAC-\angle ACD]\]

\[=90^o+\angle ACD-\dfrac{\angle AOD}{2}=90^o\]

Điều này nghĩa là $[O]$ và $[DEF]$ trực giao.

Nghịch đảo đường tròn $[DEF]$ thì $PQ$ thành $[O'PQ]$ còn $[O]$ vẫn là $[O]$. Mà $[O]$ tiếp xúc $PQ$ nên $[O]$ tiếp xúc $[O'PQ]$.

______________________________________

P.S: Không phải là chuyện gì to tát khi không ai thích tính toán trong hình học cả nhưng có buồn khi thấy phương pháp tọa độ bị phân biệt đối xử. Mình thỉnh thoảng sẽ vẫn tham gia giải bài, tuy nhiên đề nghị Bảo xóa tên mình khỏi bảng điểm đi.

Đề nghị bài tiếp theo

$\boxed{\text{Bài toán 35}}$[Quan sát] Cho $A$, $B$, $C$, $P$. $PA$, $PB$, $PC$ cắt $[PBC]$, $[PCA]$, $[PAB]$ tại $D$, $E$, $F$. $I$, $I_a$, $I_b$, $I_c$ là tâm nội tiếp, bàng tiếp $\triangle ABC$. $P^*$ đẳng giác với $P$ trong $\triangle BAC$. Khi đó $[P^*I_aD]$, $[P^*I_bE]$, $[P^*I_cF]$, $[I_aEF]$, $[I_bFD]$, $[I_cDE]$ đồng quy.

______________________________________

Bài 35 đã được post trong một bài viết của mình trên blog. Tuy có lời giải ngắn[không sơ cấp] nhưng nó lại được xem là hệ quả trực tiếp của kết quả không sơ cấp lắm. Post lên đây hi vọng các bạn quan tâm và có lời giải sơ cấp.

Một bài toán gần giống đã được thầy Hùng đăng tại //artofproblems...1174125p5654683

Mình lấy bài này là hệ quả từ cái khác nên cũng không biết lời giải sơ cấp của nó có khó không.

Trong trường hợp vấn đề quá khó thì mình khuyến khích các bạn đăng ý tưởng để cùng nhau giải quyết.

$$\begin{array}{| l | l |} \hline Ngockhanh99k48 & 6\\ \hline IHateMath & 1\\ \hline fatcat12345 & 5\\ \hline dogsteven & 5\\ \hline baopbc & 8\\ \hline QuangDuong12011998 & 4\\ \hline xuantrandong & 2\\ \hline mrjackass & 1\\ \hline vietnaminmyheart & 3\\ \hline BuiBaAnh & 1\\ \hline halloffame & 2\\ \hline\end{array}$$

Hình gửi kèm


Bài viết đã được chỉnh sửa nội dung bởi baopbc: 15-06-2016 - 10:40

  • manhhung2013, quanghung86, baopbc và 1 người khác yêu thích

Đã gửi 11-06-2016 - 15:02

Theo mình không nên phân biệt đối xử với tọa độ, đặc biệt là tọa độ Barycentric vì không phải vô cớ mà tờ báo hình sơ cấp lớn nhất thế giới  //forumgeom.fau.edu/, đã nhiều năm qua dành thời lượng lớn để nghiên cứu về loại tọa độ này. Hơn nữa thực chất những việc biến đổi tỷ số trên đường thẳng, rồi những hệ thức của hàng điểm điểu hòa đều có gốc gác là đại số và tọa độ xạ ảnh. Mặt khác cũng không phải không có lý do mà Descartes được xếp vào một trong những nhà toán học vĩ đại nhất nhờ phương pháp của mình. Do đó nên dung hòa giữa đại số và hình.


  • manhhung2013, canhhoang30011999, Bui Ba Anh và 6 người khác yêu thích

Đã gửi 14-06-2016 - 02:06

Topic hơi gián đoạn do bài toán 35 của Dương khá khó, tạm gác lại bài đó, mình xin đề xuất một bài tiếp như sau

$\boxed{\text{Bài toán 36.}}$ Cho tam giác $ABC$ cân tại $A$ nội tiếp đường tròn $[O]$ với $AD$ là đường kính của $[O]$. Các điểm $E,F$ lần lượt thuộc $DC,DB$. $G$ thuộc $EF$ sao cho $\frac{GF}{GE}=\frac{FB}{CE}$. Chứng minh rằng $CG$ và $AF$ cắt nhau trên $[O]$.

Khi đề xuất bài mình nghĩ nên có một số thứ tự các tiêu chí ưu tiên như sau

1] Đúng

2] Hay, đẹp

3] Mới

4] Sát với thi Olympic

5] Kiến thức đơn giản

6] Đề bài ngắn gọn, dễ hiểu

7] Nhiều hướng tiếp cận, tổng quát.


Bài viết đã được chỉnh sửa nội dung bởi baopbc: 14-06-2016 - 09:19

  • Zaraki, manhhung2013, canhhoang30011999 và 4 người khác yêu thích

Đã gửi 14-06-2016 - 09:03

$\boxed{\text{Lời giải bài toán 36}}$ [Bài này chắc phải dùng chút lượng giác quá!

]

$X\equiv AE\cap [O],Y\equiv AF\cap [O],G'\equiv BX\cap CY$. Theo định lí $Pascal$ thì $G'$ thuộc $EF$.

Ta có biến đổi tỉ số $\frac{FB}{EC}=\frac{\sin \angle BAY.AF}{\sin \angle CAX.AE}=\frac{\sin \angle G'CB.AF}{\sin \angle G'BC.AE}=\frac{G'B.AF}{G'C.AE}$

$K\equiv AX\cap CY,L\equiv AY\cap BX$. Do tam giác $ABC$ cân tại $A$ nên $LKXY$ là tứ giác nội tiếp $\Rightarrow LK\parallel BC$

Theo định lí $Thales$ thì $\frac{G'B}{G'C}=\frac{G'L}{G'K}$. Áp dụng định lí $Sin$ cho các tam giác $G'LF$ và $G'KE$ ta được:

$\frac{G'F}{G'E}=\frac{G'L.\sin \angle G'LF.\sin \angle AFE}{G'K.\sin \angle G'KE.\sin \angle AEF}=\frac{G'B.\sin \angle AFE}{G'C.\sin \angle AEF}=\frac{G'B.AF}{G'C.AE}=\frac{FB}{EC}\Rightarrow G\equiv G'$ hay $AF,CG$ cắt nhau trên $[O].\blacksquare$

Hình vẽ bài toán

Bài toán đề nghị. $\boxed{\text{Bài toán 37}}$ [Romani TST 2016 Day 4 Problem 2] Cho tam giác $ABC,CA\neq CB.D,F,G$ theo thứ tự là trung điểm $AB,AC,BC$. Một đường tròn $\gamma $ qua $C$, tiếp xúc với $AB$ tại $D$ và cắt $AF,BG$ lần lượt tại $H,I.H',I'$ lần lượt đối xứng với $H,I$ qua $F,G.H'I'$ cắt $CD$ và $FG$ lần lượt tại $Q,M$ và $CM$ cắt $\gamma $ tại $P$. Chứng minh rằng $\triangle CPQ$ cân.

$$\begin{array}{| l | l |} \hline Ngockhanh99k48 & 6\\ \hline IHateMath & 1\\ \hline fatcat12345 & 5\\ \hline dogsteven & 5\\ \hline baopbc & 9\\ \hline QuangDuong12011998 & 4\\ \hline xuantrandong & 2\\ \hline mrjackass & 1\\ \hline vietnaminmyheart & 3\\ \hline BuiBaAnh & 1\\ \hline halloffame & 2\\ \hline\end{array}$$


Bài viết đã được chỉnh sửa nội dung bởi baopbc: 15-06-2016 - 10:40

Đã gửi 14-06-2016 - 09:58

Cám ơn Bảo, lời giải của thầy
 

Gọi đường tròn ngoại tiếp tam giác $DEF$ cắt $[O]$ tại $S$ khác $D$. $EF$ cắt $BC$ tại $T$. Các tam giác $\triangle SFB\sim\triangle SEC$ do đó $\frac{SF}{SE}=\frac{BF}{CE}=\frac{GF}{GE}=\frac{TF}{TE}$ đẳng thức cuối có do định lý Menelaus cho tam giác $DEF$ với $T,B,C$ thẳng hàng. Từ đó ta có hàng $[EF,GT]=-1$. Gọi $M$ là trung điểm $BC$, $CF,CG$ cắt $DM$ tại $P,Q$. Chiếu xuyên tâm $C$ suy ra hàng $[DQ,PM]=C[DQ,PM]=[EF,PM]=-1$ hay $[MP,DQ]=-1$ suy ra $[MD,PQ]=1-[MP,DQ]=2$ vậy $\frac{PM}{PD}=2\frac{QM}{QD}\quad [1]$. 

Áp dụng định lý Menelaus cho tam giác $DBM$ với $C,Q,F$ thẳng hàng suy ra $\frac{FB}{FD}.\frac{QD}{QM}.\frac{CM}{CB}=1$ suy ra $2\frac{QM}{QD}=\frac{FB}{FD}\quad [2]$.

Từ [1],[2] suy ra $\frac{PM}{PD}=\frac{FB}{FD}$, vậy $PF\parallel BC$. Gọi $CP$ giao cắt $[O]$ tại $H$ khác $C$. Suy ra $\angle FPH=\angle BCH=\angle BDH$ suy ra từ giác $HFPD$ nội tiếp. Suy ra $\angle PHF=\angle PDF=\angle ADB=\angle CHA$ vậy $A,H,F$ thẳng hàng. Ta có điều phải chứng minh.


  • baopbcNo Moniker thích

Đã gửi 15-06-2016 - 09:42

$\boxed{\text{Lời giải bài toán 37}}$ [Thực sự cảm ơn Bảo nhé

]

Ta có $CH'.CA=AH.AC=AD^2=BD^2=BI.BC=CI'.CB$ nên $H', I', G, F$ đồng viên, do đó $\widehat{I'H'C}=\widehat{CGF}$. $DF, DG$ cắt $\gamma$ tại $R, S$. Ta sẽ chứng minh $R, S$ cùng thuộc $H'I'$. 

Chú ý rằng $FH'.FA=FH.FC=FR.FD$ nên $A, H', D, R$ đồng viên, do đó $\widehat{RH'F}=\widehat{FDA}=\widehat{CGF}=\widehat{I'H'C}$, do đó $R, H', I'$ thẳng hàng. Tương tự ta sẽ suy ra $R, H', Q, I', S$ thẳng hàng.

Ta có $\widehat{RSD}=\widehat{RDA}=\widehat{DFG}$, nên $R, S, G, F$  đồng viên. Do đó $MH'.MI'=MF.MG=MR.MS=MP.MC$, do đó $C,P, I', H'$ đồng viên. Gọi đó là đường tròn $\omega$. 

Chú ý rằng $\widehat{H'CQ}=\widehat{SDC}=\widehat{SRC}$ và $\widehat{QCI'}=\widehat{CDR}=\widehat{CSR}$, nên $\triangle CH'Q \sim \triangle RCQ$ và $\triangle CI'Q \sim \triangle SCQ$, do đó $QH'.QR=QC^2=QI'.QS$.

Cuối cùng, xét phép nghịch đảo: $I_{Q}^{QC^2}$ biến $C \leftrightarrow C$, $S \leftrightarrow I'$, $R \leftrightarrow H'$, do đó $[CSR] \leftrightarrow [CH'I']$, mặt khác $\{ C, P \} = [CH'I'] \cap [CSR]$ và $C$ là điểm bất động nên $P$ cũng là điểm bất động. Do đó $CQ=QP$ ta có đpcm. 

$$\begin{array}{| l | l |} \hline Ngockhanh99k48 & 7\\ \hline IHateMath & 1\\ \hline fatcat12345 & 5\\ \hline dogsteven & 5\\ \hline baopbc & 9\\ \hline QuangDuong12011998 & 4\\ \hline xuantrandong & 2\\ \hline mrjackass & 1\\ \hline vietnaminmyheart & 3\\ \hline BuiBaAnh & 1\\ \hline halloffame & 2\\ \hline\end{array}$$


Bài viết đã được chỉnh sửa nội dung bởi baopbc: 15-06-2016 - 10:41

  • quanghung86baopbc thích

Đã gửi 15-06-2016 - 11:09

$\boxed{\text{Lời giải bài toán 35}}$

Bổ đề [hiển nhiên]:  Cho $\triangle ABC$ và $\triangle DEF$. Khi đó $[AEF], [BFD], [CDE]$ đồng quy $\Leftrightarrow$ $[BCD], [CAE], [ABF]$ đồng quy. 

Áp dụng kết quả: //www.artofprob...nity/c6h1174125, ta có $[DI_bI_c], [EI_cI_a], [FI_aI_b]$ đồng quy. Theo bổ đề trên ta có: $[I_aEF], [I_bDF], [I_cDE]$ đồng quy tại $T$. Do tính đối xứng nên ta chỉ cần chứng minh $T \in [P^*I_aD]$.

Ta có $\triangle AI_cD \stackrel{+}{\sim} \triangle AP^*I_b$, nên $[I_bP^*, I_bC] \equiv [I_bP^*, I_bA] + [I_bA, I_bC] \equiv [DI_c, DA] + [AI, CI]$ [mod $\pi$]. Do đó $[I_cC, I_cD] + [I_bP^*, I_bC] \equiv [CI, I_cD] + [DI_c, DA] + [AI, CI] \equiv [AI, AD]$ [mod $\pi$]. 

Ta lại có $\triangle CDI \stackrel{+}{\sim} \triangle CI_cE$ và $\triangle CDP^* \stackrel{+}{\sim} \triangle CFE$ nên $[I_cE, I_cC] + [FC, FE] \equiv [DI, DC] + [DC, DP^*] \equiv [DI, DP^*]$ [mod $\pi$].

Mặt khác, $\triangle CFI_a \stackrel{+}{\sim} CI_bP^*$ và $\triangle AID \stackrel{+}{\sim} \triangle AP^*I_a$ $\Leftrightarrow$ $[I_bP^*, I_bC] \equiv [FI_a, FC]$ [mod $\pi$] và $[P^*I_a, P^*A] \equiv [ID, IA]$ [mod $\pi$].

Kết hợp lại ta có $[P^*I_a, P^*D] \equiv [P^*I_a, P^*A] + [P^*A, P^*D] \equiv [ID, IA] + [P^*A, P^*D] \equiv [AP^*, AI] + [DI, DP^*] \equiv [AI, AD] + [DI, DP^*] \equiv [[I_cC, I_cD] + [I_bP^*, I_bC]] + [[I_cE, I_cC] + [FC, FE]] \equiv [[I_cE, I_cC] + [I_cC, I_cD]] + [[FI_a, FC] + [FC, FE]] \equiv [I_cE, I_cD] + [FI_a, FE] \equiv [TE, TD] + [TI_a, TE] \equiv [TI_a, TD]$ [mod $\pi$]. Do đó $T \in [P^*I_aD]$, tương tự ta có đpcm. 

P/S: Bài 21 đã có lời giải ở đây ://artofproblem...h614826p6455571 của SalaF nên Bảo không cần ghim lại đâu.

Đề nghị $\boxed{\text{Bài toán 38 [AoPS]}}$. Cho $\triangle ABC$ ngoại tiếp $[I]$ và điểm Lemoine $K$. Trung trực $BC$ cắt $IB, IC$ thứ tự tại $A_b, A_c$. Định nghĩa tương tự với $B_a, B_c, C_a, C_b$. Chứng minh $K$ thuộc trục đẳng phương của $[A_bB_cC_a]$ và $[A_cB_aC_b]$.  

$$\begin{array}{| l | l |} \hline Ngockhanh99k48 & 8\\ \hline IHateMath & 1\\ \hline fatcat12345 & 5\\ \hline dogsteven & 5\\ \hline baopbc & 9\\ \hline QuangDuong12011998 & 4\\ \hline xuantrandong & 2\\ \hline mrjackass & 1\\ \hline vietnaminmyheart & 3\\ \hline BuiBaAnh & 1\\ \hline halloffame & 2\\ \hline\end{array}$$


Bài viết đã được chỉnh sửa nội dung bởi baopbc: 15-06-2016 - 11:15

  • quanghung86, viet nam in my heartbaopbc thích

Đã gửi 16-06-2016 - 18:12

Lời giải bài 38.

Bài 38 được Telv Cohl đề nghị trong marathon hình học trên AoPS //www.artofprob...1198358p5979646

và được bạn Trịnh Huy Vũ tổng quát lên cho hai điểm đẳng giác tại đây //artofproblems...onjugate_points

Mình post chứng minh bằng tọa độ barycentric trong post của Trịnh Huy Vũ.

Theo mình đây là một bài toán rất đẹp, không chỉ ở hình vẽ mà ngay cả biểu thức tọa độ. Tính toán khá nhẹ nhàng vì tọa độ các điểm khá gọn. Quá trình làm mất khoảng 1 tiếng 

$$\begin{array}{| l | l |} \hline Ngockhanh99k48 & 8\\ \hline IHateMath & 1\\ \hline fatcat12345 & 5\\ \hline dogsteven & 5\\ \hline baopbc & 9\\ \hline QuangDuong12011998 & 5\\ \hline xuantrandong & 2\\ \hline mrjackass & 1\\ \hline vietnaminmyheart & 3\\ \hline BuiBaAnh & 1\\ \hline halloffame & 2\\ \hline\end{array}$$


Bài viết đã được chỉnh sửa nội dung bởi baopbc: 20-06-2016 - 18:22

  • quanghung86, viet nam in my heartbaopbc thích

Video liên quan

Chủ Đề